LSAT and Law School Admissions Forum

Get expert LSAT preparation and law school admissions advice from PowerScore Test Preparation.

User avatar
 Dave Killoran
PowerScore Staff
  • PowerScore Staff
  • Posts: 5852
  • Joined: Mar 25, 2011
|
#86002
Complete Question Explanation
(The complete setup for this game can be found here: lsat/viewtopic.php?f=303&t=7341)

The correct answer choice is (E)

As established during the latter part of the setup discussion, N or S must attend the first class, and thus answer choice (E) is correct.
 studyhelp20
  • Posts: 28
  • Joined: Dec 09, 2020
|
#82412
Dear Power Score Staff,

Could you please provide an explanation of the correct answer for this question? Along with explanations of the incorrect answers? Thanks for the help.

Sincerely,
Brennan
 Rachael Wilkenfeld
PowerScore Staff
  • PowerScore Staff
  • Posts: 1358
  • Joined: Dec 15, 2011
|
#82511
Hi Brennan,

This question asks us for who could be first. Based on rule 3, we know that K is the first female, but NOT the first student. That means the first student must be a male. Immediately, that knocks out answer choice (A). We can knock out answer choice (B) because L is always in a group with I (a female) and so the IL group must go after K---not in slot 1. We can eliminate answer choice (E) fairly quickly because of rule number 5. If O is AFTER G, O cannot be first. Finally, we can eliminate answer choice (D) because we know based on rule number 4 that P must be after G. That leaves us with answer choice (C) as the only option.

Another way to think about this problem would be to think about who could be in the first space before looking at the answer choices. This is my preferred method--it's a type of prephrasing that makes it much faster to attack these games questions. By knowing what the answer choice is going to be before checking them, I only really have to do the work once, up front, instead of having to check answer choice by answer choice.

In this case, I started with the idea that none of the female students could be in the first slot. That leaves our potential options as LNOPS. L is with I, so now we have NOPS. O and P both are described as after another variable, which means our only potential first sloters are either N or S. Of those, S is the only one listed as an answer choice. I would select it and move along.

Hope that helps.

Get the most out of your LSAT Prep Plus subscription.

Analyze and track your performance with our Testing and Analytics Package.